subject
Physics, 18.11.2020 16:50 tchloe448

A flat loop of wire consisting of a single turn of cross-sectional area 8.00 cm2 is perpendicular to a magnetic field that increases uniformly in magnitude from 0.500 G to 1.60 M in 0.99 s. What is the resulting induced current if the loop has a resistance of 1.20 Ω?

ansver
Answers: 3

Other questions on the subject: Physics

image
Physics, 22.06.2019 04:30, 2005AngelMoore
Determine el momento angular de un rueda de 3000g moviendose a 10m/s con un radio de giro de 400cm
Answers: 2
image
Physics, 22.06.2019 04:50, andrewsaul04
The position of a crate sliding down a ramp is given by x=(0.05t^3) m, y=(1.7t^2) m, z=(6−0.85t^5/2) m, where t is in seconds. (a) determine the magnitude of the crate's velocity when t = 2 s. (b) determine the magnitude of the crate's acceleration when t = 2 s.
Answers: 2
image
Physics, 22.06.2019 09:30, abdominguez7187
An electric clothes dryer has a resistance of 8 ohms. it draws 30 a of current. what is the voltage, in volts, of the wall outlet that it is plugged into?
Answers: 2
image
Physics, 22.06.2019 11:00, sudotoxic
Engineers find a new metal that is stronger than steel but much lighter. this material is also significantly cheaper than what is currently used for most aircraft, available in large quantities, and easy to manufacture. the engineers are excited because this new material will lower the costs of buying and operating airplanes for companies. what is probably the best step for the engineers to make next?
Answers: 2
You know the right answer?
A flat loop of wire consisting of a single turn of cross-sectional area 8.00 cm2 is perpendicular to...

Questions in other subjects: